Musculoskeletal System 2

Pataasin ang iyong marka sa homework at exams ngayon gamit ang Quizwiz!

Which diagnostic test is used to confirm osteomalacia? 1 X-ray 2 Quantitative ultrasound (QUS) 3 Dual-energy x-ray absorptiometry (DXA) 4 Magnetic resonance imaging (MRI) scan

1 X-rays reveal ribbons of bone decalcification, confirming the presence of the disease. QUS and DXA are used to measure the bone mineral density in patients with osteoporosis. An MRI scan is not required, because x-rays detect the bone defects. Text Reference - p. 1553

What percentage of patients with systemic lupus erythematosus have butterfly rash? 1 10 percent 2 30 percent 3 50 percent 4 70 percent

3 Butterfly rash over the cheeks and bridge of the nose is a classic feature of systemic lupus erythematosus (SLE); it is seen in 50 percent of patients. Butterfly rash is not seen in 10 percent, 30 percent, or 70 percent of patients with SLE. Text Reference - p. 1582

As a part of discharge teaching, the nurse has explained to a patient with osteoarthritis about the disease and its treatment. Which statements of the patient indicate the need for explaining the concepts again? Select all that apply. 1 "I'll never use a staircase." 2 "I can skip doses if I don't have any symptoms." 3 "I should maintain a healthy body weight." 4 "My bones have become weak and fragile." 5 "I should not stand or kneel for a long time."

, , The use of a staircase is permitted, but using it repeatedly may be deleterious. Skipping a medication dose when there are no symptoms is not advisable. The patient should take the medication as prescribed by the health care provider. The patient should be taught that osteoarthritis involves joints and not the bones, unlike osteoporosis. Body weight is to be regulated to prevent a heavy load on the joints. The misconception that there should be no body activity should be cleared up. A balance should be maintained in rest and activity, neither of which should be prolonged. Text Reference - p. 1564

A patient is prescribed teriparatide. What information is accurate for the nurse to include in the patient's education related to medication frequency and route of administration? 1 Once a week, orally 2 Once a day, intramuscularly 3 Once a day, subcutaneously 4 Once a month, intravenously

3 Teriparatide needs to be administered subcutaneously. It is administered as a dose of one injection a day. It cannot be administered orally, intramuscularly, or intravenously. Text Reference - p. 1557

A patient with joint pain will undergo joint aspiration. The nurse explains the procedure to the patient. Arrange the sequence of joint aspiration in correct order. 1. Insert needle through stretched skin to anesthetise the area. 2. Aspirate, clean, and apply bandage. 3. Insert 18-gauge or larger needle into the joint. 4. Wash skin with povidone-iodine solution. 5. Examine the knee for amount of joint fluid.

Correct 1. Examine the knee for amount of joint fluid. Correct 2. Wash skin with povidone-iodine solution. Correct 3. Insert needle through stretched skin to anesthetise the area. Correct 4. Insert 18-gauge or larger needle into the joint. Correct 5. Aspirate, clean, and apply bandage In joint aspiration, the knee is first examined to determine the amount of joint fluid present and to check for overlying cellulitis or coexisting pathology in the joint or surrounding tissues. The skin is then washed with povidone-iodine solution, and a needle is inserted through stretched skin to anesthetize the area. An 18-gauge or larger needle is then inserted into the joint. The joint is then aspirated, the needle is removed, and a bandage is applied at the site of puncture to negate the chances of infection. Text Reference - p. 1579

Gottron's papules image

The image shows Gottron's papules. These are violet or erythematous papules and small plaques that may develop over the distal interphalangeal or metacarpophalangeal joint areas, and elbow or knee joints in about 70 percent of patients with dermatomyositis (DM). Chronic gout is characterized by multiple joint involvement and visible deposits of sodium urate crystals (tophi). Sclerodactyly is the term used for semiflexed fingers due to tightening of the skin of the wrist. It is found in patients with scleroderma. Rheumatoid nodules appear subcutaneously as firm, nontender, granuloma-type masses and are usually located over the extensor surfaces of joints, such as fingers and elbows. Text Reference - p. 1589

What integumentary change does the nurse expect to find in a patient with suspected systemic lupus erythematosus (SLE)? 1 Alopecia 2 Dysphagia 3 Splenomegaly 4 Cognitive impairment

1 Alopecia is an integumentary manifestation of SLE. Dysphagia is a gastrointestinal disorder found in SLE patients. Splenomegaly is a hematologic disorder of SLE patients. Cognitive impairment is due to neurologic disturbances. Text Reference - p. 1583

Which finding is due to Raynaud's phenomenon in a patient with suspected systemic lupus erythematosus (SLE)? 1 Cyanosis of fingers 2 Swan neck deformity of the fingers 3 Persistent lesions and photosensitivity 4 Rashes over the cheeks and bridge of the nose

1 In patients with SLE, there is symmetric pallor and cyanosis of fingers. This occurs due to Raynaud's phenomenon. Swan neck deformity of the fingers is a lupus-related arthritis or a musculoskeletal problem of SLE. Persistent lesions and photosensitivity along with mild systemic disease occur in subacute cutaneous lupus syndrome. Rash over the cheeks and bridge of the nose is a classic feature of SLE. It is called butterfly rash. Text Reference - p. 1585

Which disease is caused by Borrelia burgdorferi? 1 Lyme disease 2 Septic arthritis 3 Reactive arthritis 4 Psoriatic arthritis

1 Lyme disease is an infection caused by the spirochete Borrelia burgdorferi. Septic arthritis can be caused by any bacteria, but the most common causative organisms are Staphylococcus aureus and Streptococcus hemolyticus. Reactive arthritis is precipitated after exposure to specific genitourinary or gastrointestinal infections. Chlamydia trachomatis is responsible for causing sexually transmitted reactive arthritis. Psoriatic arthritis (PsA) is a progressive inflammatory disease whose exact cause is unknown. Text Reference - p. 1578

Which statement is correct about systemic lupus erythematosus (SLE)? 1 SLE occurs mostly in women of childbearing age. 2 Females are less likely to develop SLE than males. 3 Whites are more prone to SLE than Asian Americans. 4 The incidence of SLE in United States is approximately 4 to 5 per 10,000.

1 Most cases of SLE occur in women in their childbearing years and during and after pregnancy. Women are 10 times more prone to SLE than men. Asian Americans are more likely to develop SLE than whites. The overall incidence of SLE in the United States is 2 to 8 per 100,000. Text Reference - p. 1582

The nurse is caring for a patient with osteoarthritis. The nurse concludes the patient has accurate knowledge when the patient describes the condition as: 1 Degeneration of cartilage in synovial joints 2 An underproduction of synovial fluid 3 Breakdown of tissue caused by a sedentary lifestyle 4 An autoimmune disorder that is hereditary in nature

1 Osteoarthritis is a degeneration or breakdown of the articular cartilage in synovial joints. The condition also has been referred to as degenerative joint disease. Osteoarthritis does not affect synovial fluid, is unrelated to a sedentary lifestyle, and is not mediated by an autoimmune response. Text Reference - p. 1562

A patient has osteoarthritis of the knees. What finding would the nurse expect on examination of the patient's knees? 1 Crepitus upon movement of knee joints 2 Reddened, swollen affected joints 3 Swan-neck deformity in affected joints 4 Stiffness that is worse in the morning

1 Osteoarthritis is characterized predominantly by joint pain and crepitus on movement. Stiffness worse in the morning, swollen joints, and swan neck deformity are consistent with rheumatoid arthritis. Text Reference - p. 1562

What is the priority nursing intervention(s) for a patient recently admitted with osteomyelitis? 1 Immobilize the affected area and administer pain medications. 2 Ambulate the patient every four hours to decrease the risk of clot formation. 3 Teach the patient about the side effects of antimicrobial therapy. 4 Perform passive range of motion exercises every two hours to reduce swelling.

1 Pain management and immobilization of the extremity are the most critical nursing interventions. The patient is on strict bed rest and should not ambulate. Although range of motion and side effects are important, they are not the priority. Text Reference - p. 1542

Which is a reversible side effect of antimalarial agents used to treat systemic lupus erythematosus (SLE) patients? 1 Retinopathy 2 Nephropathy 3 Lymphadenopathy 4 Peripheral neuropathy

1 Retinopathy can develop due to high doses of antimalarial drugs but the condition usually reverses when these drugs are discontinued. Nephropathy is a renal complication that occurs in SLE and is not a side effect of antimalarial drugs. Lymphadenopathy is a hematologic disorder associated with SLE and is not a side effect of antimalarial drugs. Peripheral neuropathy is a neurologic condition associated with SLE for which antimalarial drugs are not given. Text Reference - p. 1584

The nurse is caring for a patient who has difficulty breathing, along with symptoms suggestive of Raynaud's phenomenon. Which disease does the nurse suspect is the cause of the symptoms? 1 Scleroderma 2 Lyme disease 3 Sjögren's syndrome 4 Rheumatoid arthritis

1 The nurse suspects that the patient has scleroderma, which involves internal organs and can cause abnormalities in the lung such as pleural thickening, pulmonary fibrosis, and pulmonary function abnormalities. This can cause breathing difficulties. Raynaud's phenomenon is also associated with scleroderma. Lyme disease is associated with erythema migrans, or a bull's eye lesion, at the site of a tick bite. Sjögren's syndrome involves the salivary and lacrimal glands. Rheumatoid arthritis (RA) is a chronic, systemic autoimmune disease characterized by inflammation of connective tissue in the synovial joints. Text Reference - p. 1590

Which drug may need to be discontinued in a patient receiving medication for systemic lupus erythematosus (SLE) who develops psychosis? 1 Corticosteroids 2 Antimalarial agents 3 Immunosuppressants 4 Nonsteroidal antiinflammatory agents

1 If a person with SLE develops psychosis, it may be due to the disease itself or may be due to side effects of corticosteroids. Hence, corticosteroids may need to be discontinued. Some patients with SLE may not tolerate antimalarial drugs but these drugs do not cause psychosis. Long term use of corticosteroids is reduced with immunosuppressants. Nonsteroidal antiinflammatory drugs are used for polyarthritis or polyarthralgias. These drugs may cause gastrointestinal and renal effects, not psychosis. Text Reference - p. 1586

What instruction should the nurse provide to a patient with scleroderma that experiences dysphagia and heartburn? 1 "Increase your fluid intake." 2 "Increase the amount of time between meals." 3 "Sit upright for 30 minutes after eating." 4 "Take an antacid 45 minutes before each meal."

1 In order to prevent dysphagia, the nurse must encourage the patient to drink fluids. Eating small, frequent meals also helps prevent dysphagia. Hence, the nurse should not instruct the patient to increase the amount of time between meals. In order to prevent heartburn in such patients, the nurse should instruct the patient to sit upright for at least two hours after every meal. Heartburn can also be minimized by taking antacids 45 to 60 minutes after meals, but not before meals. Text Reference - p. 1588

The nurse is reviewing proper body mechanics with a patient with a history of low back pain caused by a herniated lumbar disc. The nurse would determine proper understanding if the patient states: 1 "Physical therapy may help to alleviate the pain." 2 "I should limit exercise to one to two times per week to avoid excess strain." 3 "I should try to get out of bed in one smooth, quick motion." 4 "I will never bend down below waist level to avoid additional injury."

1 Physical therapy is recommended to improve symptom management and pain control. The patient should get out of bed by sliding to the edge of the mattress, turning onto the side, and pushing up from the bed with one or both arms. This keeps the spine in alignment and reduces strain on the lumbar spine. Exercise should not be limited, and bending down is permitted using proper body mechanics. Text Reference - p. 1547

Which medication is appropriate for a postmenopausal patient being treated for osteoporosis who is also at risk for breast cancer? 1 Raloxifene 2 Denosumab 3 Teriparatide 4 Human calcitonin

1 Raloxifene is a selective estrogen receptor modulator (SERM) that decreases the risk of breast cancer by blocking the effects of estrogen in the breast tissue. Denosumab is prescribed to postmenopausal women with osteoporosis who are at high risk of fractures. Teriparatide stimulates new bone formation and is prescribed to treat osteoporosis in men and postmenopausal women at high risk of fractures. Human calcitonin is prescribed to patients who have low tolerance for bisphosphonate drugs. Text Reference - p. 1556

While assessing a patient, the nurse notes red spots on the hands, forearms, palms, face, and lips. Which condition does the nurse suspect? 1 Scleroderma 2 Lyme disease 3 Psoriatic arthritis 4 Systemic lupus erythematosus

1 Red spots on the hands, forearms, palms, face, and lips, known as telangiectasia, are found in patients with scleroderma. In Lyme disease, there is a bull's eye lesion at the site of the tick bite. In psoriatic arthritis, there are red, irritated, scaly patches that involve the joints. In systemic lupus erythematosus, there is a typical butterfly rash over the cheeks and bridge of the nose. Text Reference - p. 1587

The nurse is admitting a patient to the nursing unit with a history of a herniated lumbar disc and low back pain. In completing a more thorough pain assessment, the nurse should ask the patient if which action aggravates the pain? 1 Bending or lifting 2 Application of warm moist heat 3 Sleeping in a side-lying position 4 Sitting in a fully extended recliner

1 Bending or lifting Back pain that is related to a herniated lumbar disc often is aggravated by events and activities that increase the stress and strain on the spine, such as bending or lifting, coughing, sneezing, and lifting the leg with the knee straight (straight leg-raising test). Application of moist heat, sleeping position, and ability to sit in a fully extended recliner do not aggravate the pain of a herniated lumbar disc. Text Reference - p. 1545

The primary health care provider asks the nurse to prepare a patient for serum analysis of uric acid levels. Which condition does the nurse anticipate in the patient? 1 Gout 2 Scleroderma 3 Osteoarthritis 4 Rheumatoid arthritis

1 Gout Gout is characterized by increased levels of uric acid in blood. Serum uric acid analysis is used in the diagnosis of gout. Scleroderma is associated with inflammatory changes in the skin, blood vessels, and internal organs. A skin or visceral biopsy is required to rule out scleroderma. Osteoarthritis is associated with inflammation of the minor joints and a physical exam, radiologic analysis like computerized tomography (CT) and x-ray scan will help in the diagnosis. Rheumatoid arthritis is associated with inflammation of the weight bearing joints and can be ruled out by performing serology tests such as rheumatoid factor and C-reactive protein. Text Reference - p. 1577

A nurse caring for a patient with osteoarthritis instructs the patient about the various nonpharmaceutical interventions for the management of pain. Which information should the nurse include in these instructions? Select all that apply. 1 Regulation of a normal body mass index (BMI) 2 Use of paraffin baths or hot packs 3 Immobilization of the affected joint 4 Use of crutches or walker if required 5 Strenuous exercise to keep the joints functional

1, 2, 4 Osteoarthritis is usually caused by a known event or condition that directly damages cartilage or causes joint instability. Pain management in osteoarthritis involves regulation of a normal body mass index, because an increase in weight may pose stress on the joints. Paraffin baths or hot packs may be used to relieve the associated pain by reducing inflammation. The use of assistive devices like a walker or crutches can ease mobility while avoiding undue excessive pressure on the affected joint. The affected joint should not be immobilized; however, strenuous activities of the joint should be avoided. Strenuous exercise can worsen the situation, and so rest should be taken during periods of acute inflammation. Text Reference - p. 1564

A patient is treated with glucosamine and chondroitin for the treatment of rheumatoid arthritis. What care should be exercised with the use of these medications? Select all that apply. 1 Do not use with warfarin 2 Use in asthmatic patients with caution. 3 Do not use in patients allergic to shellfish. 4 Monitor the blood glucose of diabetic patients. 5 Discontinue use immediately if effect is not seen.

1, 2, 4 Both glucosamine and chondroitin can be suggested to patients who are unable to take celecoxib or other NSAIDs to provide some relief for moderate to severe arthritic pain but not for mild arthritic pain. They have been found to increase the blot-clotting effect of warfarin, thereby increasing the risk of bleeding. Not much data is available about the safety of these drugs for asthmatic patients, and careful administration is advised. Glucosamine is found to raise the blood glucose level, and monitoring glucose levels is desirable. Although glucosamine is obtained from shells of crabs and lobsters, it is considered safe for patients allergic to shellfish. The drug should be used consistently over 90 to 120 days to get the desired effect; the drug should not be discontinued immediately if no effect is seen. Text Reference - p. 1567

A nurse is instructing a group of caregivers about the home care of a patient with rheumatoid arthritis. What are the instructions that the nurse should include in the teaching? Select all that apply. 1 Perform aquatic exercise in warm water. 2 Never place pillows below the knees. 3 Ensure complete bed rest for patients. 4 Apply heat for at least 30 to 60 minutes. 5 Use lifts and elevators instead of the stairs.

1, 2, 5 The goals of rheumatoid arthritis therapy include satisfactory pain management and minimal loss of functional ability of the affected joints. Aquatic exercise in warm water helps in pain management. It helps relieve joint stiffness and allows the patient to perform daily activities more comfortably. Good body alignment while resting can be maintained, and to decrease the risk of joint contracture, pillows should never be placed under the knees. A small, flat pillow may be used under the head and shoulders. Lifts and elevators should be used rather than stairs to help protect the affected joint. Patients with RA should alternate between rest and activity regimens. Heat application should not exceed 20 minutes at a time, because it may aggravate the condition. Text Reference - p. 1576

A patient is diagnosed with scleroderma. What should the nurse anticipate in the pharmacologic treatment plan? Select all that apply. 1 Bosentan 2 Reserpine 3 Leflunomide 4 Methotrexate 5 Cyclophosphamide

1, 2, 5 Scleroderma, or systemic sclerosis, is a disorder of connective tissue characterized by fibrotic, degenerative, and occasionally inflammatory changes in the skin, blood vessels, synovium, skeletal muscle, and internal organs. Bosentan may help prevent and treat digital ulcers while improving exercise capacity and heart and lung dynamics. Reserpine increases blood flow to fingers. Immunomodulators such as cyclophosphamide are immunosuppressants and inhibit DNA, RNA, and protein synthesis. Leflunomide and methotrexate, however, have not been found effective in the treatment and are not prescribed. Text Reference - p. 1588

The nurse identifies a nursing diagnosis of "impaired physical mobility related to low back pain" for a patient who has a herniated lumbar disc. What are appropriate nursing interventions for this diagnosis? Select all that apply. 1 Apply heat to the affected area. 2 Maintain a body mass index (BMI) of 18.5 to 24.9. 3 Sleep on the abdomen or in a supine position. 4 Lean forward without bending at the knees. 5 Sleep on a side-lying position.

1, 2, 5 The nurse should encourage the patient to maintain a healthy body weight, apply heat or ice to the painful area, and sleep in a side lying position to help alleviate pain and improve mobility. Sleeping on the abdomen will hyperextend the back and increase pain and decrease mobility. Bending should occur at the waist while bending the knees to support the lower back. Text Reference - p. 1548

A patient with Paget's disease is encouraged to wear a brace to relieve back pain and provide support when in the upright position. What should the nurse instruct this patient to do? Select all that apply. 1 Attend physical therapy. 2 Avoid the use of bed board. 3 Avoid falls and subsequent fractures. 4 Examine the skin for damage due to friction. 5 Lift light weights using good body mechanics.

1, 3, 4 The nurse should encourage the patient to attend physical therapy appointments to increase muscle strength. The patient should take necessary precautionary measures to avoid falls and subsequent fractures. The patient should examine the skin for damage due to friction caused by the brace. The patient should also use a firm mattress or a bed board for sleeping. The patient should avoid lifting weights to prevent fractures and further damage to the bones. Text Reference - p. 1557

Septic arthritis requires prompt treatment to prevent joint destruction and bone loss. Which nursing interventions would help achieve this goal? Select all that apply. 1 Explain need of antibiotics to patient. 2 Ensure adherence to corticosteroid therapy. 3 Support the patient who requires operative drainage. 4 Assist in early identification of the causative organism. 5 Assess and monitor joint inflammation, pain, and fever. 6 Suggest use of resting splints to immobilize affected joints.

1, 3, 4, 5, 6 Septic arthritis is caused by microorganisms invading the joint cavity. The early identification of the microorganism is achieved by blood cultures. In order to meet the goal of the therapy, the patient adherence to antibiotic therapy is vital. During the course of therapy, joint inflammation, pain, and fever should be monitored. The use of resting splints to immobilize affected joints during disease flare-ups protects the joint. In case of sepsis, the patient should be supported in operative drainage. Corticosteroids are not used in the management of septic arthritis. Text Reference - p. 1579

A nurse is assessing a patient with rheumatoid arthritis. What are the questions that should be a part of this assessment? Select all that apply. 1 Are you allergic to aspirin? 2 Have you ever been hospitalized? 3 Have you undergone any joint surgery? 4 Do you feel stiffness in your joints? 5 Have you ever had a blood transfusion? 6 Are you taking any medications currently?

1, 3, 4, 6 A nurse assesses a patient to discover important health information like past health history, current medications, sensitivity to medications, history of surgery, and functional health patterns, which is necessary in devising a safe and effective treatment plan. Details about drug allergy, joint surgery, joint stiffness, and medications are relevant to an assessment pertaining to arthritis. The questions about blood transfusion and hospitalization are not relevant to an assessment related to this condition. Text Reference - p. 1574

The nurse is caring for a patient with Duchenne muscular dystrophy. What should be included in the teaching plan for this patient? Select all that apply. Correct 1 Teach use of orthotic jacket. 2 Allow frequent inactive periods. Correct 3 Teach range-of-motion exercises. 4 Ensure patient receives prolonged bed rest. Correct 5 Recommend genetic testing for family members.

1, 3, 5 The nurse should teach the patient and caregiver the need for orthotic jacket to provide stability and prevent further deformity. The nurse should teach them range-of-motion exercises to keep the patient active as long as possible. The nurse should recommend genetic testing and counseling for family members with a family history of muscular dystrophy. The patient should be taught to limit sedentary periods to prevent skin breakdown and respiratory complications. The nurse should ensure the patient avoids prolonged bed rest, because immobility can lead to further muscle wasting. Text Reference - p. 1544

The nurse is caring for a patient following lumbar fusion. What nursing interventions should the nurse perform for this patient? Select all that apply. 1 Assess extremity circulation. 2 Use a soft mattress for comfort. 3 Place patient in Fowler's position. 4 Report severe headache to the surgeon. 5 Monitor peripheral neurologic signs.

1, 4, 5 The nurse caring for the patient who had a lumbar fusion should assess extremity circulation using temperature, capillary refill, and pulses. The nurse must report to the surgeon the presence of severe headache, which could be due to leakage of cerebrospinal fluid (CFS). The nurse must monitor the patient's peripheral neurologic signs. The nurse should perform these assessments every two to four hours during the first 48 hours after surgery. The patient should be provided with a firm mattress or bed board. The patient should be placed in supine position with pillows under the thighs of each leg or in a side-lying position with pillows between the legs for comfort and alignment. Text Reference - p. 1550

A patient has been prescribed diclofenac sodium. What precautions should be taken with its use? Select all that apply. 1 Avoid exposure to sunlight. 2 Antacids should be used concomitantly. 3 Sunscreen should be used concomitantly. 4 Avoid external heat or occlusive dressings. 5 Do not coadminister with oral aspirin.

1, 4, 5 Diclofenac sodium is a nonsteroidal antiinflammatory drug. Its gel form is used topically on the affected joints. When using the drug, exposure to sunlight should be avoided to prevent ultraviolet ray-induced skin tumors (reported in animals). Also the use of external heat or occlusive dressing can lead to burns. Coadministration of oral aspirin should not be done due to the potential for increased side effects. Concomitant use of sunscreen lotion should be avoided, because it may result in skin reactions or change the absorption of the drug. Antacids are generally used for oral NSAIDs and not as topical formulations. Text Reference - p. 1567

The nurse is caring for an elderly patient that is being treated for Paget's disease. Which medications does the nurse expect to find in the patient's prescription? Select all that apply. 1 Calcitonin 2 Raloxifene 3 Denosumab 4 Teriparatide 5 Bisphosphonates

1, 5 Calcitonin and bisphosphonates are used in the treatment of Paget's disease. Postmenopausal women use raloxifene in the treatment of osteoporosis, which also reduces the risk of breast cancer. Denosumab is prescribed for patients with osteoporosis. Teriparatide stimulates new bone formation and is prescribed to treat osteoporosis in men and postmenopausal women at a high risk for fractures. Text Reference - p. 1557

Which is the most likely cause of death in patients with scleroderma? 1 Lung disease 2 Renal diseases 3 Sjögren's syndrome 4 Gastrointestinal disturbances

1In patients with scleroderma, lung involvement is the main cause of death. It includes plural thickening, pulmonary fibrosis, and pulmonary function abnormalities. Renal diseases were previously the major cause of death in diffuse scleroderma. However, newer improvements in dialysis, bilateral nephrectomy in patients with uncontrollable hypertension, and kidney transplantation offer hope for many patients with renal failure. Angiotensin-converting enzyme inhibitors also help treat the renal disease associated with scleroderma. Patients with scleroderma may develop Sjögren's syndrome. It is associated with xerostomia and dryness of the eyes, but not death. Gastrointestinal (GI) disturbances in scleroderma patients include esophageal fibrosis and constipation. Text Reference - p. 1587

A patient is hospitalized with diabetes mellitus and a herniated lumbar disc. Which breakfast choice is most appropriate for this patient? 1 White toast and orange juice 2 Bran cereal and milk 3 Egg-white omelet 4 Protein shake

2 Each meal should contain at least one source of fiber, which will reduce the risk of constipation and straining at stool, which increases back pain. Bran, a high-fiber food, is appropriate for selection from the menu. White toast, egg whites, and a protein shake are not high in fiber; the white toast and orange juice also may elevate the patient's blood sugar. Text Reference - p. 1548

A patient has bilateral osteoarthritis of the knees. The nurse teaches the patient that the most beneficial measure to protect the joints is to: 1 Use a wheelchair to avoid walking as much as possible 2 Exercise regularly and maintain a well-balanced diet 3 Use a cane for ambulation to relieve the pressure on the hips 4 Avoid sitting at a 90-degree angle or full flexion of the knees

2 Because maintaining an appropriate load on the joints is essential to the preservation of articular cartilage adaptation, the patient should maintain an optimal overall body weight or lose weight if overweight. Avoiding walking is contraindicated; exercise should continue to promote mobility and relieve pain. A cane may be indicated to help with mobility, but is not used to relieve hip pressure, because the osteoarthritis is in the knees. Sitting at a 90-degree angle would be indicated following hip replacement. Text Reference - p. 1564

What would the nurse plan as an appropriate short-term goal for a patient with degenerative joint disease (DJD)? 1 The patient will limit physical activity in the morning. 2 The patient will participate in physical therapy activities. 3 The patient will eliminate the use of narcotic analgesics if diarrhea develops. 4 The patient will limit pain medications to nonnarcotic drugs to prevent addiction.

2 Because pain and discomfort are major clinical manifestations of degenerative joint disease (DJD), or osteoarthritis, relief measures are the first priority. Relief can be achieved with physical therapy and other pain-management measures. Limitation of physical therapy, elimination of pain medication, and limitation of pain medication to nonnarcotic drugs are all incorrect goals for a patient with DJD. The patient needs to stay physically active and use narcotic or nonnarcotic analgesics, depending on the level of pain. Text Reference - p. 1564

A patient who is being treated for osteoporosis receives a prescription for a bisphosphonate drug. What instruction should the nurse give to the patient regarding its administration and aftereffects? 1 "You may feel like throwing up and lose your appetite." 2 "You should drink a full glass of water with the medicine." 3 "You should lie down immediately after eating, for 15 minutes." 4 "You should eat a meal within 15 minutes after taking the medicine."

2 Bisphosphonate drugs should be administered correctly to facilitate absorption into the body. The drug should be consumed with a full glass of water. The patient may experience flu-like symptoms such as fever and headache without any loss of appetite or vomiting. After eating, the patient should remain upright for 30 minutes and should not lie down. The medicine should be taken 30 minutes before having food. Text Reference - p. 1556

What is common to both polymyositis (PM) and dermatomyositis (DM)? 1 Both increase a patient's risk for cancer. 2 Both have clinical manifestations of joint redness and pain. 3 Both show the same pathologic findings on a muscle biopsy. 4 Both may cause Gottron's sign at the elbow or knee joints.

2 Both polymyositis (PM) and dermatomyositis (DM) may cause joint inflammation which is manifested as joint redness and pain. Patients with DM have an increased risk for cancer, whereas such association is not seen with PM. A muscle biopsy of both PM and DM shows necrosis, degeneration, regeneration, and fibrosis. However, the pathologic findings are distinct for both DM and PM. Gottron's sign is present only in patients with DM, not in those with PM. Text Reference - p. 1589

What is the reason that estrogen therapy is not considered to be a primary treatment for preventing osteoporosis in postmenopausal women? 1 Weight loss and gastritis 2 Breast and uterine cancer 3 Leg cramps and hot flushes 4 Nasal dryness and irritation

2 Estrogen therapy for women after menopause is not considered a primary treatment to prevent osteoporosis due to the increased risk of breast and uterine cancer. Common side effects of bisphosphonates include weight loss and gastritis, especially if not administered properly. Raloxifene mimics the effects of estrogen in a positive way, but adverse effects include leg cramps and hot flushes. Calcitonin, if administered as a nasal spray, can cause nasal dryness and irritation. Text Reference - p. 1556

Which is the most ominous sign of advanced systemic lupus erythematosus (SLE)? 1 Psychosis 2 Dysrhythmias 3 Butterfly rash 4 Polyarthralgia

2 Fibrosis of the sinoatrial and atrioventricular nodes lead to dysrhythmias, which are an ominous sign of advanced SLE, contributing significantly to morbidity and mortality. Psychosis may occur either as a neurologic manifestation of SLE when the central nervous system is involved, or as a side effect of corticosteroid therapy. Butterfly rash is a characteristic sign of SLE but can be suppressed using topical immunomodulators. Polyarthralgia is often the patient's first complaint and can be treated with the use of nonsteroidal antiinflammatory drugs. Text Reference - p. 1582

The public health nurse is planning to provide community education to increase the number of people who seek health care promptly after a tick bite. What information should the nurse provide when teaching people who are at risk for a tick bite? 1 The best therapy for the acute illness is an intravenous (IV) antibiotic. 2 Check for an enlarging reddened area with a clear center. 3 Surveillance is necessary during the summer months only. 4 Antibiotics will prevent Lyme disease if taken for 10 days.

2 Following a tick bite, the expanding "bull's eye rash" is the most characteristic symptom, which usually occurs in 3 to 30 days. There also may be flu-like symptoms and migrating joint and muscle pain. Active lesions are treated with oral antibiotics for two to three weeks and doxycycline is effective in preventing Lyme disease when given within three days after the bite of a deer tick. IV therapy is used with neurologic or cardiac complications. Although ticks are most prevalent during summer months, residents of high-risk areas should check for ticks whenever they are outdoors. Text Reference - p. 1578

A patient has just been given a diagnosis of osteoarthritis. The nurse explains that the associated changes in the joint are the result of what type of process? 1 Malignant 2 Degenerative 3 Inflammatory 4 Immunologic

2 Osteoarthritis occurs commonly after years of stress, or "wear and tear," on weight-bearing joints. This degenerative process causes hypertrophic changes in the joints. Malignant refers to a growth process that is resistant to treatment, such as that seen in cancer. Inflammatory is the term given to a nonspecific immune response that occurs in reaction to any type of bodily injury. Immunologic refers to the immune system, which protects us from or resists disease or infection as a result of the development of antibodies or cell-mediated immunity. Text Reference - p. 1562

A patient is diagnosed with osteosarcoma without metastasis. The patient wants to know why he will be given chemotherapy before the surgery. What is the best rationale the nurse should tell the patient? 1 The chemotherapy is being used to save your left leg. 2 Chemotherapy is being used to decrease the tumor size. 3 The chemotherapy will increase your five-year survival rate. 4 Chemotherapy will help decrease the pain before and after surgery.

2 Preoperative chemotherapy is used to decrease tumor size before surgery. The chemotherapy will not save his leg if the lesion is too big or there is neurovascular or muscle involvement. Adjunct chemotherapy after amputation or limb salvage has increased the five-year survival rate in people without metastasis. Chemotherapy is not used to decrease pain before or after surgery. Text Reference - p. 1543

Which features are found in systemic lupus erythematosus (SLE)? 1 Photosensitivity, weight gain, and chronic inflammation 2 Discoid erythema, altered immunity, and photosensitivity 3 Skin rash, alopecia, and decreased erythrocyte sedimentation rate 4 Photosensitivity, increased erythrocyte sedimentation rate, and leukocytosis

2 Systemic lupus erythematosus (SLE) causes round, coin-shaped lesions called discoid erythema along with altered immunity, because SLE is a multisystem inflammatory autoimmune disease. Photosensitivity is also commonly seen in SLE patients. SLE leads to weight loss rather than weight gain. SLE causes an increase in erythrocyte sedimentation rate. SLE causes leukopenia rather than leukocytosis. Text Reference - p. 1585

The nurse caring for a patient with muscular dystrophy (MD) recalls which true statement about the disease? 1 MD is curable with early and aggressive treatment. 2 MD has an insidious onset with loss of muscle strength. 3 A viral infection is often a predisposing factor associated with the disease 4 Respiratory distress is a clinical manifestation early in the disease process

2 The cause of muscular dystrophy is basically unknown, and it is currently incurable, although the manifestations are treatable. It is characterized by an insidious loss of strength affecting skeletal muscles. Muscular dystrophy is not associated with viral infections. Respiratory distress is seen in the later, not the early, stages of the disease. Muscular dystrophy is a progressive chronic disease that is not curable; however, the manifestations can be controlled with aggressive treatment and vigilance. Text Reference - p. 1544

When caring for a patient with systemic lupus erythematosus, the nurse recognizes which major and serious complication of the disorder? 1 Dysphagia 2 Renal failure 3 Peripheral neuropathies 4 Multiple open skin lesions

2 The multisystem nature of systemic lupus erythematosus (SLE) places the patient at risk for multiple complications and is ultimately fatal. Renal failure is one of the major and serious complications of SLE. Although dysphagia, peripheral neuropathies, and multiple open skin lesions may be present in the patient with SLE, they are not consistently present and are not as serious as renal failure. Text Reference - p. 1582

A patient is prescribed intranasal salmon calcitonin for osteoporosis. What should the nurse teach this patient? 1 Spray both nostrils daily. 2 The spray may cause nasal dryness. 3 Spraying at night decreases nasal irritation. 4 A common side effect of the medication is nausea.

2 The nurse should inform the patient that nasal dryness and irritation are frequent side effects of the nasal spray. The patient should not spray both nostrils daily; instead, spray alternate nostrils daily. The nasal spray does not cause nausea. Administering the dosage at night has not been shown to decrease nasal irritation. Text Reference - p. 1556

How is osteoarthritis (OA) in men different from that in women? 1 Before age 45, hip OA is more common in men than in women. 2 After age 45, knee OA is more common in women than in men. 3 Before age 50, women are affected by OA more often than men. 4 After age 50, women are affected three times more often than men.

2 After the age of 45 years, OA of the knee is more common in women than in men. Before the age of 45 years, OA of the knee, rather than that of the hip, is more common in men than in women. Before the age of 50 years, men are affected by OA more often than women. After the age of 50 years, women are affected more often than men. However, it is twice as often and not three times as often. Text Reference - p. 1562

While assessing a patient with dermatomyositis, the nurse notes that the patient has an erythematous scaling rash on the buttocks. What condition is indicated by the assessment findings as recognized by the nurse? 1 Heliotrope 2 Poikiloderma 3 Calcinosis cutis 4 Gottron's papule

2 An erythematous scaling rash on the buttocks of a patient with dermatomyositis (DM) indicates poikiloderma. Heliotrope is an erythematous symmetric rash found around the eyes of patients with DM. Calcinosis cutis are calcium nodules which may develop throughout the skin in long-standing DM. Gottron's papules are also found in patients with DM. These are violet or erythematous papules and small plaques that develop over the distal interphalangeal or metacarpophalangeal areas, and at the elbow or knee joints. Text Reference - p. 1589

Which is an effect of metabolic bone disease? 1 Overgrowth of cartilage and bone 2 Reduction in bone mass and strength 3 Symmetric wasting of skeletal muscles 4 Infection of the bone and bone marrow

2 Any dysfunction of hormones, nutrition, or genetic factors affects bone metabolism and leads to a reduction in bone mass and strength. A benign tumor is characterized by an overgrowth of mass that can develop in the bone, muscle, fat, nerve, or cartilage. Progressive weakening and symmetric wasting of skeletal muscles is a hereditary condition indicative of muscular dystrophy. Microorganisms cause infection of the bone and bone marrow but not bone deterioration, which is a marker for metabolic bone disease. Text Reference - p. 1553

Which is the best diagnostic test to confirm Paget's disease? 1 X-rays 2 Bone scan 3 Blood serum tests 4 Dual-energy x-ray absorptiometry (DXA)

2 Bone scans are the most preferred test to confirm Paget's disease. Bone scans using radiolabeled bisphosphonate demonstrate increased uptake in the affected skeletal areas. X-rays may demonstrate that the normal contour of the affected bone is curved and the bone cortex is thickened and irregular. A serum alkaline phosphatase test is effective during advanced stages of the disease. DXA is used as an alternative to x-rays. It reveals the bone mineral density in patients with bone disorder. Text Reference - p. 1557

Which is a common clinical feature found in patients with systemic lupus erythematosus? 1 Telangiectasia 2 Butterfly rash 3 Bull's eye rash 4 Kyphotic posture

2 Butterfly rash over the cheeks and bridge of the nose is a characteristic feature of systemic lupus erythematosus. Telangiectasia is a feature of scleroderma. Bull's eye rash is seen in Lyme disease. Kyphotic posture is seen in advanced ankylosing spondylitis. Text Reference - p. 1582

What is the correlation of bone resorption to bone deposition in osteoporosis? 1 Bone deposition exceeds bone resorption. 2 Bone resorption exceeds bone deposition. 3 Bone deposition is equal to bone deposition. 4 Bone resorption is followed by bone replacement.

2 In osteoporosis, bone resorption exceeds bone deposition. Osteomalacia is weakening of the bones due to bone deposition exceeding bone resorption. Normally, there is an equal rate of bone deposition to absorption. Paget's disease is a bone disorder in which excessive bone resorption is followed by the replacement of existing bone with larger, disorganized, and weaker new bone. .Text Reference - p. 1554

Which disease is characterized by a white phase, a blue phase, and a red phase? 1 Lyme disease 2 Scleroderma 3 Reiter's disease 4 Sjögren's syndrome

2 In scleroderma, Raynaud's phenomenon produces a blanching of the fingers and toes due to less blood flow (white phase). Then, hemoglobin releases oxygen to the tissues and cyanosis occurs (blue phase). This phase is followed by the occurrence of erythema (red phase). There is no Raynaud's phenomenon in Lyme disease, Reiter's disease, or Sjögren's syndrome. Text Reference - p. 1587

Which of the following refers to bone remodeling? 1 Deterioration of bone tissue 2 Deposition and formation of bone 3 Decalcification and softening of bones 4 Resorption of bone followed by replacement

2 In the human body, the process of bone formation followed by bone resorption is called remodeling. The rate of deposition equals the rate of formation in an individual without a bone disorder. The process of deterioration of bone tissue leading to increased bone fragility is known as osteoporosis. Decalcification and softening of the bones is caused by vitamin D deficiency, resulting in osteomalacia in an individual. Excessive bone resorption followed by replacement of bone with new bone, which is larger, disorganized, and weak is termed Paget's disease. Text Reference - p. 1554

When reinforcing health teaching about the management of osteoarthritis (OA), the nurse determines that the patient needs additional instruction after making which statement? 1 "I should take the naproxen as prescribed to help control the pain." 2 "I should try to stay standing all day to keep my joints from becoming stiff." 3 "I can use a cane if I find it helpful in relieving the pressure on my back and hip." 4 "A warm shower in the morning will help relieve the stiffness I have when I get up."

2 It is important to maintain a balance between rest and activity to prevent overstressing the joints with OA. Naproxen may be used for moderate to severe OA pain. Using a cane and warm shower to help relieve pain and morning stiffness are helpful. Text Reference - p. 1564

The nurse is caring for a patient admitted to the nursing unit with osteomyelitis of the tibia. Which symptom will the nurse most likely find on physical examination of the patient? 1 Nausea and vomiting 2 Localized pain and warmth 3 Paresthesia in the affected extremity 4 Generalized bone pain throughout the leg

2 Osteomyelitis is an infection of bone and bone marrow that can occur with trauma or surgery, or it can spread from another part of the body. Because it is an infection, the patient will exhibit typical signs of inflammation and infection, including localized pain and warmth. Nausea, vomiting, and paresthesia of the extremity are not expected to occur. Pain occurs, but it is localized, not generalized throughout the leg. Text Reference - p. 1540

During a follow-up visit, the nurse observes a tumor-like overgrowth in the bone of a patient being treated for Paget's disease. Which complication does the nurse suspect? 1 Endochroma 2 Osteosarcoma 3 Ewing's sarcoma 4 Chondrosarcoma

2 Patients with Paget's disease face the risk of developing osteosarcoma, which is a primary malignant bone tumor. It affects the long bones of the leg and pelvis, similar to Paget's disease. Endochroma, Ewing's sarcoma, and chondrosarcoma are other types of bone tumors that are not related to Paget's disease. Text Reference - p. 1557

Which criterion is used by the National Osteoporosis Foundation to recommend treatment of osteoporosis in postmenopausal women? 1 T-score of -2.5 2 T-score of below -2.5 3 T-score of -1 or higher 4 T-score between -1 and -2.5

2 The criterion used by the National Osteoporosis Foundation to recommend treatment of osteoporosis in postmenopausal women is a T-score of below -2.5, which increases the severity of osteoporosis. A T-score of -2.5 confirms osteoporosis. An individual with a T-score of -1 or higher has normal bone density. T-score between -1 and -2.5 is considered to have osteopenia, because the bone loss is more than normal, but not enough to term it osteoporosis. Text Reference - p. 1555

A patient who underwent spinal surgery reports a severe headache afterward. When changing the dressing to the incisional area, the nurse finds the drainage to be slightly yellow, and positive on a dipstick test. What is the best nursing intervention in this situation? 1 Send the drainage sample for culture. 2 Inform the primary health care provider. 3 Administer an analgesic for the headache. 4 Apply a new dressing on the incisional area.

2 The presence of yellowish drainage on the dressing after spinal surgery, along with patient reports of a headache, may indicate leakage of cerebrospinal fluid. Therefore, the nurse should inform the primary health care provider to determine the supportive interventions. The drainage sample does not necessarily need to be sent for culture, because there are no other signs of infection such as a fever. Analgesics are administered to treat the headache; however, they will not help to resolve the cause of the headache. A new dressing should be applied to the incisional area, if the dressing is soiled. However, this intervention may not help in preventing cerebrospinal fluid leakage. Text Reference - p. 1550

Which stage of rheumatoid arthritis is characterized by extensive muscle atrophy and joint deformities such as subluxation? 1 Early stage 2 Severe stage 3 Terminal stage 4 Moderate stage

2 The severe stage of rheumatoid arthritis is characterized by extensive muscle atrophy, and joint deformities such as subluxation, ulnar deviation and osteoporosis. The early stage of rheumatoid arthritis is characterized by possible x-ray evidence of osteoporosis, but no destructive changes on x-ray. The terminal stage of rheumatoid arthritis is characterized by fibrous or bony ankylosis and adjacent muscle trophy. A patient with moderate rheumatoid arthritis will experience adjacent muscle atrophy, possible presence of extraarticular lesions and osteoporosis with or without bone destruction. Text Reference - p. 1569

The nurse is providing information to a patient with Paget's disease about the positive effects of physical therapy. Which activity does the nurse suggest to the patient? 1 Running 2 Walking 3 Twisting and turning 4 High-impact aerobics

2 Walking Physical therapy increases muscle strength and ensures good body mechanics. Walking should be suggested as the preferred exercise. The patient should be discouraged to take up running because it may put too much stress on the bones. High-impact exercise involving twisting and turning or aerobics should be avoided. Text Reference - p. 1557

What drugs may help treat pulmonary arterial hypertension in patients with scleroderma? Select all that apply. 1 Lisinopril 2 Bosentan 3 Nifedipine 4 Tetracycline 5 Omeprazole

2, 3 Bosentan and nifedipine are used in the treatment of pulmonary hypertension for patients with scleroderma. Bosentan helps to decrease pulmonary resistance, and nifedipine acts as a vasodilator. Lisinopril is used to treat renal diseases in such patients. Tetracycline is used to treat diarrhea. Omeprazole, a proton pump inhibitor, is used to treat esophageal symptoms. Text Reference - p. 1587

A patient is diagnosed with myalgic encephalomyelitis. What are the signs and symptoms that the nurse is likely to find in this patient? Select all that apply. 1 Refreshing sleep 2 Sore throat 3 Impaired memory 4 Improved work output 5 Headache

2, 3, 5 Myalgic encephalomyelitis, also known as chronic fatigue syndrome, is a disorder characterized by debilitating fatigue and a variety of associated complaints. A patient with myalgic encephalomyelitis experiences sore throat, impaired memory, and headache due to the fact that it affects the central nervous system and hypothalamic-pituitary-gonadal (HPG) axis. The patient always feels tired. The patient does not get refreshing sleep, and thus work output is also very low. Text Reference - p. 1592

The nurse is caring for a pediatric patient who has Duchenne muscular dystrophy. Which treatment strategies would be beneficial for the child? Select all that apply. 1 Performing limb salvage 2 Fitting for an orthotic jacket 3 Administering hyperbaric oxygen 4 Prescribing corticosteroid therapy 5 Recommending shock-absorbing shoe inserts

2, 4 A patient with muscular dystrophy has a risk of spinal collapse due to progressive weakening of the neck muscles. Fitting for an orthotic jacket provides support to the muscles and decreases the risk of spinal collapse. Corticosteroid therapy maintains airways and improves respiratory functioning. Limb salvage is prescribed for the patient who has osteosarcoma. Hyperbaric oxygen is used as adjuvant therapy in the patient with chronic osteomyelitis. Shock-absorbing shoe inserts helps to prevent back pain, especially in females. Text Reference - p. 1544

What is suggested as etiologies of chronic fatigue syndrome (CFS)? Select all that apply. 1 Trauma 2 Microorganisms 3 Environmental factors 4 Neuroendocrine abnormalities 5 Changes in the central nervous system

2, 4, 5 Many microorganisms such as cytomegaloviruses and Epstein-Bar viruses have been found to be associated with chronic fatigue syndrome (CFS). Neuroendocrine abnormalities such as hypothalamus pituitary axis and hypothalamus pituitary gonadal axis malfunction have been found in CFS. Both regulate the pituitary and gonadal level response. In some patients, it is found that there is decreased memory and lack of concentration; hence changes in CNS can be seen. Trauma may serve as a trigger in susceptible persons and lead to fibromyalgia, rather than CFS. Environmental factors may be important for development of systemic lupus erythematosus and spondyloarthropathies but not CFS. Text Reference - p. 1592

A patient observes discoloration of urine during the course of the treatment for arthritis. How does the nurse explain this type of observation? Select all that apply. 1 It may be due to an infection. 2 Excess medication is excreted in the urine. 3 This is a side effect of the treatment. 4 This is a result of the interaction between the medications with the urine. 5 It will stop once the therapy is discontinued.

2, 4, 5 Many drugs often cause discoloration of the urine because of the conversion into colored metabolites (metabolism by mostly liver enzymes and not interaction with urine) in the body, which are then excreted in the urine. The discoloration of urine by the drug is due to the excess medicine being excreted in the urine, and the resultant interaction between the drug metabolite and the urine. This side effect vanishes with the discontinuation of the therapy. Infections generally do not cause discoloration. Discoloration of urine is not a side effect of the drug. Text Reference - p. 1566

A primary health care provider has prescribed an intraarticular injection of steroids for a patient. Which statement holds true for this type of administration? Select all that apply. 1 It is only used for short-term therapy. 2 The joint should not be overused after injection. 3 It cannot be accompanied with systemic therapy. 4 Sterile technique must be used for steroid injections. 5 A dose of medication is delivered to the affected joint.

2, 4, 5 The intraarticular dose of steroids is injected directly into the joint space of a painful, inflamed arthritic joint and is often used as an adjunct to systemic therapy. Overuse of the joint is not suggested in the first few hours after an injection, because it can aggravate arthritis. A sterile technique must be used for steroid injections in order to reduce the risk of infection. Generally, a dose of medication is injected, and the effect lasts a week or month. Intraarticular steroid injections are a part of long-term therapy and are used as an adjunct to systemic therapy. Text Reference - p. 1578

What should a nurse teach a patient who has systemic lupus erythematosus (SLE)? Select all that apply. 1 "Use a powder to keep the rash dry." 2 "Consider using heat therapy for joint pain." 3 "Avoid going out in the sun from noon to 2 PM." 4 "Use sunscreens with sun protection factor greater than 15." 5 "Plan an appointment for pregnancy counseling."

2, 4, 5 The nurse should teach the patient with SLE to consider using heat therapy for joint pain or arthralgia. The patient should use sunscreens that have a sun protection factor (SPF) of at least 15. SLE can cause infertility, so the patient should go for pregnancy counseling before planning a pregnancy. The nurse should teach the patient to avoid powders. Patients should be instructed to avoid going out in the sun between 11 AM and 3 PM. Text Reference - p. 1586

What information would be appropriate for a nurse to include in educational materials on the primary risk factors for osteoporosis in women? Select all that apply. 1 Alcohol use 2 Family history 3 Cigarette smoking 4 Low fluoride intake 5 Early bone reabsorption 6 Decreased calcium intake over the lifespan

2, 4, 6 The primary risk factors for osteoporosis in women include advanced age, small frame, family history, low lifetime calcium intake, inactive lifestyle, and low fluoride and vitamin D intake. Other factors associated with osteoporosis, but not as significant, are alcohol use, smoking, and early bone reabsorption. Text Reference - p. 1562

The nurse is assessing the recent health history of a patient with osteoarthritis. The nurse determines that the patient was managing the condition well when the patient states that their activity pattern has consisted of which of the following? 1 Minimal exercise with frequent rest periods 2 Bed rest and walking to the restroom 3 Walking and swimming regularly 4 Running three miles most days of the week

3 A regular low-impact exercise such as walking is important in helping to maintain joint mobility in the patient with osteoarthritis. Exercise should continue to improve symptom management and decrease pain. Bed rest is not warranted and high impact exercise, such as running, may worsen pain. Text Reference - p. 1564

The nurse is caring for a patient who has osteoarthritis (OA) of the knees. The nurse teaches the patient that the most beneficial measure to protect the joints is to engage in what activity? 1 Use a wheelchair to avoid walking as much as possible. 2 Sit in chairs that cause the hips to be lower than the knees. 3 Eat a well-balanced diet to maintain a healthy body weight. 4 Use a walker for ambulation to relieve the pressure on the hips.

3 Because maintaining an appropriate load on the joints is essential to the preservation of articular cartilage integrity, the patient should maintain an optimal overall body weight or lose weight if overweight. Walking is encouraged. The chairs that would be best for this patient have a higher seat and armrests to facilitate sitting and rising from the chair. Relieving pressure on the hips is not important for OA of the knees. Text Reference - p. 1564

What intervention should the nurse consider while caring for a patient with fibromyalgia? 1 Administering long-acting opioids 2 Avoiding administering any sedatives 3 Encouraging the patient to reduce the intake of coffee and alcohol 4 Advising the patient to completely avoid any stretching and exercise

3 Coffee and alcohol can act as irritants to muscles. Hence, the nurse should encourage the patient with fibromyalgia to reduce the intake of coffee and alcohol. Long-acting opioids are generally not given unless fibromyalgia is refractory to other therapies. Sedatives, such as benzodiazepines, can be prescribed. Stretching and exercise can reduce muscle tension and spasm and can be performed under the guidance of a physical therapist. Text Reference - p. 1592

Which medication is contraindicated in a patient with a history of multiple fractures who has osteoporosis? 1 Calcitonin 2 Raloxifene 3 Corticosteroids 4 Bisphosphonates

3 Corticosteroids should be used with extreme caution in patients with osteoporosis. Calcitonin therapy is advised in patients who have low tolerance for bisphosphonates drugs. Raloxifene is a selective estrogen receptor modulator (SERM) that decreases the risk of breast cancer in a menopausal patient with osteoporosis. Treatment with bisphosphonates is considered for patients who are already being treated with corticosteroids. Text Reference - p. 1554

Which assessment finding would alert the nurse to the presence of osteoarthritis in a patient admitted to the hospital after a fall? 1 Joint stiffness that is worse in the morning 2 Swan neck deformities 3 Crepitus with knee movement 4 A measurable loss of height

3 Crepitus over joints is suggestive of osteoarthritis. A gradual but measurable loss of height and the development of kyphosis, or "dowager's hump," is indicative of the presence of osteoporosis; a swan neck deformity and stiffness that is worse in the morning occur with rheumatoid arthritis. Text Reference - p. 1562

Which are the precipitating or aggravating factors of systemic lupus erythematosus (SLE)? 1 Azathioprine, hydralazine, and procainamide 2 Procainamide, chloroquine, and sun exposure 3 Sun exposure, hydralazine, and oral contraceptives 4 Cyclophosphamide, oral contraceptives, and azathioprine

3 In SLE patients, the onset or exacerbation of disease symptoms sometimes occurs due to sun exposure. SLE may also be precipitated or aggravated by certain drugs, such as hydralazine and oral contraceptives. Azathioprine is used as a treatment drug in SLE. Procainamide can also precipitate SLE. However, chloroquine is used to treat fatigue and moderate skin and joint problems in patients with SLE. Cyclophosphamide and azathioprine are used to treat SLE. Text Reference - p. 1582

Which is the correct statement regarding scleroderma? 1 It is found more often in the elderly population. 2 It is present with asymmetrical painful swellings. 3 It is associated with tissue fibrosis and occlusion of blood vessels. 4 It is more common in Caucasian patients than in African American patients.

3 In scleroderma, there is an increased production of collagen. This leads to increased tissue fibrosis and results in the occlusion of blood vessels. It is more common in patients between the ages of 30 and 50 years. Typical lesions are symmetrical and painless. Scleroderma is found more often in African Americans as compared to Caucasians. Text Reference - p. 1587

The nurse is admitting a patient who has a new onset of lower back pain. To differentiate between the pain of a lumbar herniated disc and lower back pain from other causes, what would be the best question for the nurse to ask the patient? 1 "Is the pain worse in the morning or in the evening?" 2 "Is the pain sharp or stabbing, or burning or aching?" 3 "Does the pain radiate down the buttock or into the leg?" 4 "Is the pain totally relieved by analgesics, such as acetaminophen (Tylenol)?"

3 Lower back pain associated with a herniated lumbar disc is accompanied by radiation along the sciatic nerve and commonly can be described as traveling through the buttock, to the posterior thigh, or down the leg. This is because the herniated disc causes compression on spinal nerves as they exit the spinal column. Time of occurrence, type of pain, and pain relief questions are important, but they do not elicit differentiating data. Text Reference - p. 1548

A patient diagnosed with osteomalacia is prescribed nutritional therapy and phosphorus supplements. Which other nonpharmacologic instruction is appropriate for the nurse to include on the patient's care plan? 1 Instructing the patient to wear a corset 2 Instructing the patient to use a firm mattress 3 Encouraging the patient to expose self to sunlight 4 Encouraging the patient to perform high-impact aerobics

3 Osteomalacia is caused by vitamin D deficiency, which can be helped by exposure to sunlight and ultraviolet rays. In Paget's disease, the patient may be required to wear a corset or light brace to relieve back pain and provide support when in an upright position. Along with the corset, a firm mattress should be used to provide back support and relieve pain. The patient with osteomalacia should be discouraged from high-impact aerobics and activities such as running, lifting, and twisting, because they put too much stress on the bones. Text Reference - p. 1553

Which outcome is expected from the administration of zoledronic acid in the treatment of Paget's disease? 1 Increased bone mass 2 Decreased hypocalcaemia 3 Decreased bone resorption 4 Decreased breast cancer risk

3 Patients with Paget's disease are prescribed a bisphosphonate, zoledronic acid, which decreases bone resorption. Teriparatide increases the action of osteoblasts, thereby increasing the bone mass. Calcium and vitamin D supplements help decrease hypocalcaemia, a common side effect of drugs prescribed for Paget's disease. Raloxifene is effective in decreasing the risk of breast cancer in postmenopausal women with osteoporosis. Text Reference - p. 1556

Which disease is treated with pilocarpine? 1 Scleroderma 2 Lyme disease 3 Sjögren's syndrome 4 Systemic lupus erythematosus

3 Pilocarpine is a cholinergic drug that stimulates the production of saliva. It is used in the treatment of Sjögren's syndrome in which dryness of the mouth occurs. Vasoactive agents are prescribed to treat scleroderma. In Lyme disease, antibiotics are prescribed to treat early-stage infection and prevent later stages of the disease. In systemic lupus erythematosus, nonsteroidal antiinflammatory and immunosuppressive drugs are prescribed. Text Reference - p. 1590

A patient has a low-grade fever, conjunctivitis, and arthritic pain in the long joints, which is asymmetric. The patient informs the nurse that he or she had a genitourinary infection in the recent past. From what is the patient most likely suffering? 1 Rheumatoid arthritis 2 Gout 3 Reactive arthritis 4 Psoriatic arthritis

3 Reactive arthritis is a condition that is often found to be triggered by genitourinary infection. The symptoms are complex, including fever, conjunctivitis, and joint pain. The pain is usually asymmetric. Rheumatoid arthritis, gout, and psoriatic arthritis do not involve asymmetric joint pain, and none of these includes conjunctivitis as a symptom of the disease. Text Reference - p. 1581

A patient comes into the clinic requesting a blood test for Lyme disease. The nurse knows that: 1 Most cases occur along the southeastern and gulf coast area of the United States (US). 2 More than 50,000 new cases are reported every year in the US. 3 A bull's eye rash occurs in 70% to 80% of infected patients at the site of the tick bite. 4 Lyme disease symptoms can mimic those of systemic lupus erythematosis (SLE).

3 Seventy percent to eighty percent of patients diagnosed with Lyme disease develop erythema migrans (EM), which is a skin lesion that appears at the site of the tick bite within 3 and 30 days after exposure. Most cases appear along northeastern US, midwestern states of Minnesota and Wisconsin, and the northwestern coast. About 20,000 cases are reported annually. SLE is a chronic, multisystem inflammatory autoimmune disease that affects the skin, joints, and serous membranes and follows an unpredictable course. Text Reference - p. 1578

A patient's complex symptomatology over the past year has led to a diagnosis of systemic lupus erythematosus (SLE). Which statement demonstrates the patient's need for further teaching about the disease? 1 "I'll try my best to stay out of the sun this summer." 2 "I know that I probably have a high chance of getting arthritis." 3 "I'm hoping that surgery will be an option for me in the future." 4 "I understand that I'm going to be vulnerable to getting infections."

3 Surgery is not a key treatment modality for SLE, so this indicates a need for further teaching. SLE carries an increased risk of infection, sun damage, and arthritis. Text Reference - p. 1585

A patient has an erythematous symmetric rash and swelling around the eyes. The nurse assesses red, smooth patches on the hands sparing the interphalangeal spaces. Which disease does the nurse suspect? 1 Polymyositis 2 Lyme disease 3 Dermatomyositis 4 Systemic lupus erythematosus

3 The erythematous symmetric rash around the eyes, called heliotrope, and edema around the eye suggests dermatomyositis. The symmetrically distributed red, smooth, or scaly patches that spare the interphalangeal spaces indicate Gottron's sign, which also suggests dermatomyositis. In polymyositis, there is no rash formation. In Lyme disease, a bull's eye lesion is found at the site of the tick bite. In systemic lupus erythematosus , a classic butterfly rash is found over the cheeks and bridge of the nose. Text Reference - p. 1589

The nurse determines that dietary teaching for a patient with osteoporosis has been successful when the patient selects which highest calcium meal? 1 Chicken stir-fry with 1 cup each onions and green peas, and 1 cup of steamed rice 2 Ham and Swiss cheese sandwich on whole wheat bread, steamed broccoli, and an apple 3 A sardine (3 oz) sandwich on whole wheat bread, 1 cup of fruit yogurt, and 1 cup of skim milk 4 A two-egg omelet with 2 oz of American cheese, one slice of whole wheat toast, and a half grapefruit

3 The highest calcium content is present in the lunch containing milk and milk products (yogurt) and small fish with bones (sardines). Chicken, onions, green peas, rice, ham, whole wheat bread, broccoli, apple, eggs, and grapefruit each have less than 75 mg of calcium per 100 g of food. Swiss cheese and American cheese have more calcium, but not as much as the sardines, yogurt, and milk. Text Reference - p. 1555

A patient with fever has symptoms of fatigue, joint pain, and stiffness. Which disease condition would the nurse suspect the patient to have? 1 Gout 2 Osteoarthritis 3 Rheumatoid arthritis 4 Ankylosing spondylitis

3 The nurse should suspect rheumatoid arthritis in this patient, since systemic manifestations such as fatigue and fever are not present in osteoarthritis, gout, and ankylosing spondylitis. Text Reference - p. 1569

On the assessment of a new admission, the nurse finds the toes and fingers to be a bluish hue accompanied with tingling and numbness. What should the nurse record the condition as? 1 Psoriatic arthritis 2 Reactive arthritis 3 Raynaud's phenomenon 4 Subacute cutaneous lupus

3 The observation is Raynaud's phenomenon, in which patients have diminished blood flow to the fingers and toes on exposure to cold due to paroxysmal vasospasm of the digits. The color changes are often accompanied by numbness and tingling. Reactive arthritis (Reiter's syndrome) is associated with a symptom complex that includes urethritis, conjunctivitis, and mucocutaneous lesions. Psoriasis is characterized by red, irritated, and scaly patches. Subacute cutaneous lupus is a dermatologic manifestation of systemic lupus erythematosus (SLE) with lesions, photosensitivity, and mild systemic disease. Text Reference - p. 1587

A patient with gout should be assessed by the nurse for which complication? 1 Cirrhosis 2 Gastric ulcer 3 Renal calculi 4 Pulmonary emboli

3 Uric acid, an end product of purine metabolism, and may exceed the kidneys' excretion capacity, resulting in the formation of a uric acid renal calculus. Gout and increased serum uric acid level do not cause cirrhosis, gastric ulcer, or pulmonary emboli. Text Reference - p. 1577

A patient with a tumor in the left leg states there is severe pain at the site of the tumor. The patient states, "Why is there so much pain?" What is the best response by the nurse? 1 "The pain medication must not be working." 2 "This is a side effect of radiation therapy." 3 "The tumor may be pressing on nerves near the bone." 4 "If one does not get enough exercise, the leg will stiffen up."

3 When the tumor presses on nerves or other organs, it causes severe pain. The question does not discuss any medications or doses that would be given for pain. Radiation therapy actually is used to help decrease the pain. Exercising will actually help decrease the pain as well. Text Reference - p. 1544

A patient is receiving alendronate for treatment of osteoporosis. The nurse would become concerned after noting which action by the patient? The patient: 1 Takes the medication with 8 ounces of water 2 Increases intake of calcium and vitamin D 3 Takes the medication immediately before bed 4 Continues to perform weight-bearing exercises

3 A patient taking alendronate should remain in an upright position, not lie down and go to bed. Alendronate is indicated for the treatment of osteoporosis and should be taken with a full glass of water. The patient should then remain in an upright position for at least 30 minutes to reduce epigastric discomfort. Taking the medication with 8 ounces of water, increased intake of calcium and vitamin D, and weight-bearing exercises are all correct actions. Text Reference - p. 1556

A patient with osteomyelitis in the left femur has been receiving gentamicin therapy for two weeks. During a follow up visit, which patient statement indicates that the treatment should be discontinued? 1 "I don't have a fever now." 2 "I have severe pain in my left leg." 3 "I have a ringing sensation in my ear." 4 "The pus from the wound has stopped draining."

3 A ringing sensation in the ear indicates ototoxicity due to the gentamicin. The treatment should be stopped to prevent worsening of the complication. Absence of fever may indicate that the infection has been reduced, but treatment should not be stopped. Severe pain in the leg is a manifestation of osteomyelitis and does not indicate that the treatment should be stopped. Absence of pus indicates effectiveness of treatment, but does not mean that the treatment should be stopped. Text Reference - p. 1541

What group of drugs is likely to have caused visual impairment in a patient being treated for systemic lupus erythematosus (SLE)? 1 Uricosuric drugs 2 Antileprosy drugs 3 Antimalarial agents 4 Antiinflammatory drugs

3 Antimalarial agents such as hydroxychloroquine, when used to treat fatigue and moderate skin and joint problems in SLE, may cause visual disturbances. Uricosuric drugs are used in the treatment of gout. Antileprosy drugs are used as an alternate therapy in those patients who cannot tolerate antimalarial agents; they are not associated with visual changes. Antiinflammatory drugs are used to treat polyarthralgias and polyarthritis. Text Reference - p. 1584

Which result of a blood serum test confirms Paget's disease? 1 Increased serum calcium level 2 Decreased serum 25-hydroxyvitamin D 3 Elevated serum alkaline phosphatase levels 4 Decreased serum calcium or phosphorus levels

3 Blood tests for Paget's disease detect increased levels of serum alkaline phosphatase indicating high bone turnover. Diagnostic tests for osteoporosis reveal an increased level of serum calcium. In patients with osteomalacia, the level of serum 25-hydroxyvitamin D is decreased. Osteoporosis reports normal levels of serum calcium, phosphorus, and alkaline phosphatase, although alkaline phosphatase may be elevated. Text Reference - p. 1557

Which distinct features of a new bone are revealed in the x-ray of a patient with Paget's disease? 1 Brittle and porous 2 Overgrowth of bone 3 Large and disorganized 4 Soft with ribbons of decalcification

3 In Paget's disease, excessive bone resorption is followed by bone replacement. The new bone is large, disorganized, and structurally weak. In osteoporosis, the bones are porous and brittle, making the patient prone to fractures. Osteochondroma reveals an overgrowth of cartilage and bone near the end of the bone at the growth plate. In osteomalacia, the bones get decalcified and softened and x-rays reveal ribbonlike decalcification of bones.

What is a typical manifestation of dermatomyositis? 1 Calcinosis 2 Butterfly rash 3 Gottron's sign 4 Bull's eye lesion

3 In dermatomyositis, reddened, smooth, or scaly patches appear with symmetric distribution sparing the interphalangeal spaces. This manifestation is known as Gottron's sign. Calcinosis is a painful deposition of calcium in the skin. It is found in patients with scleroderma. Butterfly rash is found in systemic lupus erythematosus. A bull's eye lesion is found in patients with Lyme disease. Text Reference - p. 1589

The nurse is caring for an older patient with primary osteosarcoma of the distal femur. The nurse knows that: 1 Metastasis occurs in over 50% of individuals 2 Chemotherapy is contraindicated preoperatively 3 This malignancy often is associated with prior radiation 4 This is a rare form of cancer in children and young adults

3 In the older patient, osteosarcoma is often associated with Paget's disease and prior radiation. After diagnosis, metastasis only occurs in about 10 percent to 20 percent of individuals. Chemotherapy is often given preoperatively to reduce the tumor size before surgery. Osteosarcoma is the most common cancerous bone tumor in children and young adults. Text Reference - p. 1542

A nurse is caring for a patient with a ruptured lumbar disk. What is the optimal position for this patient? 1 Lying on the back with a small pillow 2 Lying on the abdomen with no pillow 3 Lying on the side with the legs flexed 4 Lying on the side with the legs straight

3 Lying on the side with the legs flexed is the most comfortable and therapeutic position for a patient with a ruptured, or herniated, disk. This position lessens muscular stress and tension on the injured area and can help promote healing. Lying on the back with a small pillow, lying on the abdomen with no pillow, and lying on the side with the legs straight are not recommended and may increase muscle strain in the lumbar area. Text Reference - p. 1549

What diagnosis does the nurse expect after noticing Gottron's sign while assessing a patient? 1 Polymyositis 2 Scleroderma 3 Dermatomyositis 4 Systemic lupus erythematosus

3 The Gottron's sign is seen in patients with dermatomyositis. In this condition, reddened smooth or scaly patches appear in the same symmetric distribution but sparing the interphalangeal spaces. In polymyositis, there is no rash. In scleroderma, the patient often has red spot formation on the hands, palms, face, and lips (telangiectasia). In systemic lupus erythematosus, a typical butterfly rash occurs over the cheeks and bridge of the nose. Text Reference - p. 1589

Upon a patient's return from vertebral disc surgery, which of the following would the nurse report to the health care provider immediately? 1 Parasthesias 2 Constipation 3 Severe headache 4 Abdominal distention

3 The spinal cord may be entered during surgery, so a severe headache could potentially represent a cerebrospinal fluid leakage. Parasthesias, or numbness and tingling, may not be relieved immediately after surgery. It is normal for interference with bowel function to occur for several days following the surgery. Text Reference - p. 1550

When assessing a patient, the nurse notes that the fingers are fixed in a semiflexed position, with tightened skin to the wrist. Which condition is indicated by the symptoms recognized by the nurse? 1 Calcinosis 2 Heliotrope 3 Sclerodactyly 4 Telangiectasia

3 Tightening of the skin of the fingers and toes is known as sclerodactyly; it is associated with scleroderma. Calcinosis is the painful deposition of calcium on skin. Heliotrope is a violet-colored, erythematous, cyanotic rash with edema around the eyes. In telangiectasia, there are red spots on the hands, face, forehead, palms, and lips. Text Reference - p. 1587

What is the primary goal of prescribing vitamin D3 and vitamin D2 supplements for vitamin D deficiency in a patient diagnosed with osteomalacia? 1 Relief from back pain 2 Regeneration of the bones 3 Recalcification of the bones 4 Replenishment of bone mass

3 Vitamin supplements are used in the recalcification and remineralization of softened bones. Mild analgesics and antidepressants provide pain relief and comfort to a patient rather than working on bone recalcification. Teriparatide is used in bone regeneration; this drug stimulates the formation of new bones. It is used to treat osteoporosis in men and postmenopausal women at high risk for fractures. Bisphosphonates inhibit osteoclastic bone resorption and thereby replenish the bone mass and increase the bone density. Text Reference - p. 1553

How often should a dose of intravenous infusion of zoledronic acid be administered to a patient being treated for osteoporosis? 1 Each month for six months 2 Each time the patient fractures a hip 3 Annually from the time of the last infusion 4 When the after-effects of the previous dose wear off

3 Annually Zoledronic acid is a bisphosphonate drug that is used to slow bone resorption. It should be administered annually and the effect of the medicine lasts for as long as two years. It should not be administered every time there is a hip fracture. The patient may experience flu-like symptoms in the first few days after administration, but the infusion is not administered more than once a year. Text Reference - p. 1556

Which medication prescribed to a patient with hyperlipidemia can increase the risk for osteomalacia? 1 Phenytoin 2 Corticosteroid 3 Cholestyramine 4 Divalproex sodium

3 Cholestyramine Cholestyramine is used for the treatment of hyperlipidemia and can increase the risk of osteomalacia. Corticosteroids used over a long period of time may lead to osteoporosis. Divalproex sodium is an antiseizure medication that interferes with bone metabolism and may result in osteoporosis. Text Reference - p. 1553

A nurse is assessing a patient with osteoarthritis. What finding would be expected with this disorder? 1 Fever and malaise 2 Morning stiffness of all joints that tends to last all day 3 Crepitation in the weight-bearing joints 4 Increased erythrocyte sedimentation rate

3 Crepitation in the weight-bearing joints Osteoarthritis primarily affects the weight-bearing joints; therefore crepitation (a cracking or clicking sound heard in joint movement) may be present in affected individuals. Fever and malaise and increased sedimentation rate are manifestations of rheumatoid arthritis. Morning stiffness of all joints may appear in OA, but usually resolves within 30 minutes instead of lasting all day. Text Reference - p. 1562

What drugs are used to treat patients with systemic lupus erythematosus (SLE)? Select all that apply. 1 Allopurinol 2 Etanercept 3 Eculizumab 4 Prednisone 5 Cyclophosphamide

3, 4, 5 SLE is an autoimmune disorder. Eculizumab, prednisone, and cyclophosphamide are immunosuppressive agents used in the treatment of SLE. Allopurinol is a uricosuric drug used for treating gout. Etanercept is used in ankylosing spondylitis to treat inflammation and improve spinal mobility. Text Reference - p. 1583

The nurse is reviewing the report of a patient's synovial fluid analysis. Which findings would suggest the presence of rheumatoid arthritis? Select all that apply. 1 Presence of sodium urate crystals 2 Clear yellow in color 3 Presence of fibrin flecks 4 Increased matrix metallopeptidase (MMP)-3 enzyme 5 Elevated white blood cell (WBC) count

3, 4, 5 The presence of fibrin flecks in the synovial fluid indicates rheumatoid arthritis. There is also an increase in the MMP-3 enzyme, which indicates progressive joint damage. The WBC count is usually elevated up to 25,000 cells per microliters. Presence of sodium urate crystals in synovial fluid is indicative of gout. A clear yellow synovial fluid is found in osteoarthritis. Text Reference - p. 1571

Which drug should be included as part of the instructions when the nurse is teaching a patient who has lupus nephritis due to systemic lupus erythematosus (SLE)? Select all that apply. 1 Colchicine 2 Allopurinol 3 Azathioprine 4 Sulfasalazine 5 Cyclophosphamide

3, 5 Cyclophosphamide, a cytotoxic agent, is an effective treatment for lupus nephritis, which is seen as a serious complication of SLE. The nurse should teach the patient about the proper use and side effects of azathioprine, prescribed to reduce the need of corticosteroids for long-term use, and cyclophosphamide. Colchicine is an antiinflammatory drug used to treat acute attacks of gout. Allopurinol is a uricosuric drug used to prevent future attacks of gout. Sulfasalazine is a sulfa drug indicated for the treatment of rheumatoid arthritis and inflammatory bowel disease. Text Reference - p. 1584

What are the nursing interventions that should be included in the care plan for an elderly patient being treated for Paget's disease? Select all that apply. 1 Provide a soft mattress. 2 Provide an assistive device. 3 Encourage physical therapy. 4 Encourage lifting and twisting. 5 Encourage the patient to wear a corset brace.

3, 5 Physical therapy to increase muscle strength should be encouraged in patients with Paget's disease. The patient may be required to wear a corset or light brace to relieve back pain and provide support when in the upright position. A firm mattress should be used to provide back support and relieve pain. The nurse should initiate training on the use of an assistive device to prevent falls. Activities that include lifting and twisting should be discouraged. Text Reference - p. 1557

Which is the best criterion for the diagnosis of fibromyalgia? 1 Irritable bowel syndrome 2 Acute localized pain episodes 3 Biopsy of the muscles showing fiber atrophy 4 Widespread pain lasting three months or longer

4 According to the American College of Rheumatology, an individual is suffering from fibromyalgia if two criteria are met: pain is present in 11 of the 18 tender points on palpation, and there is persistent widespread pain for more than three months. Irritable bowel syndrome is common in fibromyalgia; however, it is not a manifestation specific to the condition. Acute localized pain episodes are not characteristic of fibromyalgia. Instead, there is widespread pain that occurs on both sides of the body. Muscle biopsy may show fiberatrophy but this is a nonspecific finding. Text Reference - p. 1591

What causes hypercoagulability in a patient with systemic lupus erythematosus (SLE)? 1 Lupus nephritis 2 Erythema migrans 3 Sjogren's syndrome 4 Antiphospholipid syndrome

4 Antiphospholipid syndrome causes hypercoagulability in SLE patients. It is a disorder of coagulation that leads to clots in the arteries and veins and increases the risk of stroke, gangrene, and heart attack. Lupus nephritis is a renal manifestation of SLE but does not cause hypercoagulability. Erythema migrans is a skin lesion that is a characteristic sign of Lyme disease. Sjogren's syndrome is a manifestation of rheumatoid arthritis that does not cause any coagulation. Text Reference - p. 1582

A patient with osteoarthritis has been prescribed treatment with celecoxib. What is the reason to justify this medical intervention? 1 The patient is taking warfarin. 2 The patient has severe disease condition. 3 The patient has hyperchlorhydria. 4 The patient does not tolerate traditional nonsteroidal antiinflammatory agents (NSAIDs).

4 Celecoxib is a selective cyclooxygenase-2 (COX-2) inhibitor and is devoid of the side effects of conventional NSAIDs. However, its use is limited to selected cases in which the side effects outweigh the benefits or when conventional therapy is not tolerated. Its use can be justified in a patient having severe gastrointestinal disturbances due to conventional therapy. The use of celecoxib should be carefully administered concomitantly with warfarin, because it potentiates the anticoagulant effect of warfarin. The medication is also indicated for use in mild disease conditions. Patients with hyperchlorhydia can take NSAIDs and do not need celecoxib. Text Reference - p. 1567

A nurse assesses a patient with joint pain and stiffness who was diagnosed with Stage III rheumatoid arthritis (RA). What characteristics should the nurse expect to observe? 1 Nodules present 2 Consistent muscle strength 3 Localized disease symptoms 4 Subluxation of joints without fibrous ankylosis

4 In Stage III severe RA, there may be extraarticular soft tissue lesions or nodules present and there is subluxation without fibrous or bony ankylosis. The muscle strength is decreased because there is extensive muscle atrophy. The manifestations are systemic, not localized. Text Reference - p. 1570

Which statement about Reiter's syndrome is correct? 1 It is more common in the elderly than in the young. 2 It is more common in female patients than in male patients. 3 It is most often seen as a symmetrical, bilateral type of arthritis. 4 It is most often seen in individuals who are positive for HLA-B27.

4 Individuals who are positive for the HLA-B27 gene have an increased risk of developing Reiter's syndrome, or reactive arthritis after sexual contact or gastrointestinal infection. The disease is more common in young men than in women. This type of arthritis tends to be asymmetrical rather than symmetrical and bilateral. Text Reference - p. 1581

The nurse notices malar and discoid rashes over the cheeks and nose of a patient. Which condition does the nurse suspect? 1 Scleroderma 2 Lyme disease 3 Dermatomyositis 4 Systemic lupus erythematosus (SLE)

4 Malar and discoid rashes are seen in patients with SLE. Scleroderma does not produce malar or discoid rashes. In limited disease, it may cause thickening of the facial skin. In diffuse disease, the skin loses elasticity and becomes taut and shiny, producing the typical expressionless faces with tightly pursed lips. Lyme disease is a spirochetal infection where inflammation occurs due to bite of an infected deer tick. Bull's eye rash is typical of Lyme disease. In dermatomyositis, the patient's face may show a classic violet-colored, cyanotic, or erythematous symmetric rash (heliotrope) with edema around the eyelids. Text Reference - p. 1584

Which system is involved in Raynaud's phenomenon seen in patients with systemic lupus erythematosus (SLE)? 1 Urinary 2 Gastrointestinal 3 Musculoskeletal 4 Cardiopulmonary

4 Raynaud's phenomenon involves the cardiopulmonary system. It is manifested as cyanosis of the fingers that occurs when decreased oxygen is delivered to the tissues due to vasospasm. Involvement of the urinary system in SLE patients causes glomerulonephritis, hematuria, and proteinuria. Involvement of the gastrointestinal system causes abdominal pain, diarrhea, dysphagia, nausea, and vomiting. Involvement of the musculoskeletal system causes arthritis, myositis, and synovitis. Text Reference - p. 1583

Which is an example of an autoimmune disease? 1 Gout 2 Fibromyalgia 3 Lyme disease 4 Systemic lupus erythematosus (SLE)

4 SLE is a multisystem inflammatory autoimmune disease. Gout is a type of recurring acute arthritis. It occurs due to an error in purine metabolism or secondary to inhibition of uric acid excretion. Fibromyalgia is a disorder involving neuroendocrine/neurotransmitter dysregulation. Lyme disease is a spirochetal infection caused by Borrelia burgdorferi. Text Reference - p. 1582

A patient receiving long-term nonsteroidal antiinflammatory drug (NSAID) therapy for osteoarthritis develops gastrointestinal bleeding and erosion. What should be the first intervention? 1 Dietary modifications 2 Administration of antacids 3 Switch to low-dose therapy 4 Proton pump inhibitor therapy

4 The major side effects of nonsteroidal antiinflammatory agents area gastrointestinal bleeding and ulceration, which are to be monitored during the course of the treatment. The coadministration of a proton pump inhibitor helps to treat this condition. Low-dose therapy will still be irritating to the gastric mucosa. Antacids only decrease the irritation by increasing the pH of the stomach and do not treat the bleeding. Dietary modifications are necessary to help heal the eroded surface but are not the first intervention. Text Reference - p. 1567

What should the nurse teach a patient with chronic fatigue syndrome (CFS)? 1 "Stay on complete bed rest." 2 "Avoid dark-colored fruits and vegetables in your diet." 3 "Eat a proper diet which is rich in fats and carbohydrates." 4 "Take an antiinflammatory drug if you have pain or fever."

4 The nurse should teach the patient to take nonsteroidal antiinflammatory drugs (NSAIDS) if there is pain or fever. Complete bed rest is not advised, because it may have the psychologic effect of the patient of becoming disabled. Dark-colored fruits and vegetables rich in antioxidants form an essential part of the treatment of chronic fatigue syndrome. A diet rich in fat and carbohydrates is not required. Instead, the diet should be balanced and high in fiber content. Text Reference - p. 1593

Which patient statement most clearly suggests a need to assess the patient for ankylosing spondylitis (AS)? 1 "My right elbow has become red and swollen over the last few days." 2 "I wake up stiff every morning and my knees just don't want to bend." 3 "My spouse tells me that my posture has become so stooped this winter." 4 "My lower back pain seems to be getting worse all the time and nothing seems to help."

4 AS primarily affects the axial skeleton. Based on this, symptoms of inflammatory spine pain are often the first clues to a diagnosis of AS. Knee or elbow involvement is not consistent with the typical course of AS. Back pain is likely to precede the development of kyphosis. Text Reference - p. 1580

The nurse is caring for a patient with systemic lupus erythematosus (SLE) who develops dry eyes and dry mouth. What is the most likely diagnosis? 1 Felty syndrome 2 Reiter's syndrome 3 Primary Sjögren's syndrome 4 Secondary Sjögren's syndrome

4 In secondary Sjögren's syndrome, an autoimmune disease (in this case, SLE) develops first. Sjögren's syndrome is a disorder of the immune system in which the patient's white blood cells attack the saliva and tear glands, leading to dry mouth and eyes because the body's tear and saliva production is reduced. Felty syndrome occurs in severe, nodule-forming rheumatoid arthritis (RA) and is characterized by splenomegaly and leukopenia. Reiter's syndrome, or reactive arthritis, is associated with symptoms such as urethritis, conjunctivitis, and mucocutaneous lesions. In primary Sjögren's syndrome, there is no other autoimmune disease before the dry eyes and dry mouth occurs. Text Reference - p. 1590

Which finding will help the nurse differentiate osteomalacia from osteoporosis? 1 X-Ray results 2 Serum calcium level 3 Vitamin D deficiency 4 Muscular weakness and pain

4 Muscular pain and weakness in the arms and legs is indicative of osteomalacia; whereas, osteoporosis is present when there is a bone fracture. X-rays may reveal demineralization of the bone in case of osteomalacia but x-rays are not very effective at detecting osteoporosis until more than 25 percent to 40 percent of calcium in the bone is lost due to structural deterioration. Laboratory findings indicate decreased serum calcium levels in the case of osteomalacia; the same levels are considered normal for osteoporosis. Vitamin deficiency leads to osteomalacia as well as osteoporosis. Text Reference - p. 1555

Which is a clinical manifestation of osteomalacia? 1 Dementia 2 Headaches 3 Loss of hearing 4 Weakness in the pelvic girdle

4 Muscular weakness in the pelvic girdle is a clinical manifestation of osteomalacia. This makes simple tasks such as getting up from a chair difficult. Patients with Paget's disease report dementia, headaches, and loss of hearing. Text Reference - p. 1553

A patient undergoing a blood test remarks, "I think my head is getting bigger. My hats don't fit me anymore." The nurse suspects what diagnosis? 1 Osteomalacia 2 Osteoporosis 3 Osteoarthritis 4 Paget's disease

4 Patients with Paget's disease report that their heads is becoming larger due to thickening of the skull. Osteomalacia is manifested by muscular weakness, especially in the pelvic region, which renders simple tasks such as getting up from a chair difficult. Patients with osteoporosis have weak and fragile bones, which leads to fractures. Osteoarthritis refers to pain in the joint and does not result in enlargement of the skull. Text Reference - p. 1557

A patient asks the nurse for nonsurgical methods to relieve pain caused by hammer toe. What should the nurse tell the patient? 1 Use a shoe with a stiff sole. 2 Wear shoes with wide forefoot. 3 Use longitudinal arch supports. 4 Use a metatarsal arch support.

4 The nurse should ask the patient with hammer toe to use a metatarsal arch support and perform passive manual stretching of the proximal interphalangeal (PIP) to relieve pain. Shoes with stiff soles are recommended for patients with hallux rigidus. Patients with hallux valgus, or bunions, should wear shoes with a wide forefoot to relieve pressure on the bursal sac. Patients with pes planus, or flatfoot, are asked to use resilient longitudinal arch supports to relieve pain in the foot or leg. Text reference 1552

When the patient is diagnosed with muscular dystrophy, what information should the nurse include in the teaching about this disorder? 1 Prolonged bed rest will be used to decrease fatigue 2 An orthotic jacket will limit mobility and may contribute to deformity 3 Continuous positive airway pressure (CPAP) will be used to facilitate sleeping 4 Remaining active will prevent skin breakdown and respiratory complications or distress

4 With muscular dystrophy, it is important for the patient to remain active for as long as possible. Prolonged bed rest should be avoided because immobility leads to further muscle wasting. An orthotic jacket may be used to provide stability and prevent further deformity. CPAP is used as respiratory function decreases, before mechanical ventilation is needed to sustain respiratory function and maintain activity. Text Reference - p. 1544

The registered nurse presents information to a group of student nurses about medications used to treat osteoporosis. What statement made by a student nurse indicates the need for further teaching? 1 "Alendronate is an oral tablet, taken once a week." 2 "Ibandronate is an oral tablet, taken once a month." 3 "Risedronate is an oral tablet, taken once a month." 4 "Zoledronic acid is an intravenous (IV) infusion, given once a month."

4 Zoledronic acid may be given as a bone-building drug. Zoledronic acid is approved for an annual IV infusion rather than monthly and can prevent osteoporosis for two years. Alendronate is a tablet taken orally once a week. Ibandronate and Risedronate are oral tablets taken once a month. Text Reference - p. 1556

The nurse is caring for a patient who is prescribed a muscle relaxant for acute low back pain. What should the nurse teach the patient about managing low back problems? 1 Maintain complete bed rest. 2 Sleep in prone position. 3 Use a soft comfortable mattress. 4 Place a foot on a stool during prolonged standing.

4 he nurse should tell the patient to place a foot on a stool during prolonged standing to avoid straining the lower back. The patient should avoid complete bed rest; it is better for the patient to continue daily activities. The patient should be asked to sleep in supine or side-lying positions with knees and hips flexed to prevent unnecessary pressure on support muscles, ligaments, and lumbosacral joints. A firm mattress or a bed board is recommended instead of a soft mattress. Text Reference - p. 1546

What defect does the nurse anticipate in the new bone of a patient with Paget's disease? 1 Soft bone 2 Porous bone 3 Smaller bone 4 Disorganized bone

4 Disorganized bone In Paget's disease, there is abnormal bone formation where the existing bone that is broken is replaced quickly with new bone that is disorganized and weaker than normal bone. Patients with osteomalacia experience decalcification of bones, which leads to softening of the bones. Osteoporosis is characterized by porous bones. In Paget's disease, bone tissues are replaced by new bone tissues that are larger, rather than smaller, and different in shape than the old bone. Text Reference - p. 1557

A patient, hospitalized with osteomyelitis, has a prescription for bed rest. The nurse would place the highest priority on which intervention? 1 Ambulate the patient to the bathroom every three hours. 2 Offer the patient a urinal every hour. 3 Provide activities to prevent restlessness. 4 Reposition the patient every two hours.

4 Reposition the patient every two hours. The patient is at risk for atelectasis of the lungs and skin breakdown from the prescribed bed rest. For this reason, the nurse should change the patient's position frequently to promote lung expansion and prevent pressure ulcers. If the patient is on bed rest, he or she should not ambulate to the bathroom. The patient should be offered a urinal every two hours and should be provided with activities to complete while on bed rest; however, these are not the nursing priorities. Text Reference - p. 1542

The nurse is teaching a patient in the menopausal stage to include calcium-rich food in her diet to delay the onset of osteoporosis. What should the nurse recommend to the patient as a rich source of calcium? 1 Consume an apple or a banana daily. 2 Include an egg in daily diet. 3 Consume a raw fresh carrot daily. 4 Take a cup of skim milk daily.

4 Take a cup of skim milk daily. The nurse should ask the patient to have a cup of skim milk daily, because it is a rich source of calcium. A cup of skim milk contains 302 mg of calcium. Eggs, apples, and bananas are poor sources of calcium. An egg contains only 28 mg of calcium. An apple or a banana contains only 10 mg of calcium, whereas a medium-sized carrot contains about 14 mg of calcium. Text Reference - p. 1555

Which measures reduce the risk of osteoarthritis in a patient? Select all that apply. 1 Avoiding intake of fish 2 Exercising on a hard surface 3 Increasing the intake of vitamin K 4 Avoiding bending the knee past 90 degrees 5 Avoiding forceful, repetitive movements

4, 5 Bending the knees past 90 degrees increases the risk of knee injury. Hence, maintaining an appropriate angle during exercise will prevent osteoarthritis. Forceful and repetitive movements on a hard surface may tear the ligaments and cause permanent damage. There is no reason for patients with osteoarthritis to avoid eating fish. Exercising on a soft surface will prevent injuries to the smaller joints. Vitamin K supplements reduce the risk of bleeding disorders but not the risk of osteoarthritis. Text Reference - p. 1568

Which disorders may cause peripheral neuropathy? Select all that apply. 1 Fibromyalgia 2 Osteoarthritis 3 Dermatomyositis 4 Rheumatoid arthritis 5 Systemic lupus erythematosus (SLE)

4, 5 Peripheral neuropathy, or irritation of the nerves of the extremities, may occur in SLE and rheumatoid arthritis. Fibromyalgia is a musculoskeletal disorder that does not cause neuropathy. Osteoarthritis is a localized disease with a variable, progressive course; it does not cause neuropathy. Dermatomyositis is characterized by muscle changes along with significant skin changes, but not peripheral neuropathy. Text Reference - p. 1572


Kaugnay na mga set ng pag-aaral

Grammar and vocabulary for cambridge advanced and proficiency Unit 3

View Set

General Data Protection Regulation

View Set

Anthem - Tools for Compliant Selling

View Set

Iggy Chapter 40: Concepts of Care for Patients With Problems of the Central Nervous System: The Spinal Cord

View Set

Chapter 3- Introduction to Contracts

View Set

Chapter 3: The Accounting Cycle: End of the Period

View Set

Adapted Physical Education Exam 1

View Set